women EOC

Réussis tes devoirs et examens dès maintenant avec Quizwiz!

A 25-year-old woman presents with a chief complaint of amenorrhea for three months and reports that her menstruation is usually regular. She runs approximately five miles three times per week. Her body mass index is normal. She is sexually active and uses condoms for protection. Other symptoms include fatigue and breast tenderness. What is the most likely cause of her amenorrhea?

Pregnancy ------------------------------------- Excessive exercise (A), other forms of severe stress, and severe dietary restriction can cause amenorrhea, but are not present in this patient.

A 32-year-old woman presents to your office for a physical exam including a Papanicolaou test (Pap smear). Lab results reveal negative cytology and positive human papillomavirus (HPV). Which of the following is the most appropriate next step in management?

Repeat Pap smear and human papillomavirus testing in one year

A 22-year-old woman presents with vaginal bleeding with clots that started one hour prior to arrival. She is 10 weeks pregnant based on a previous ultrasound confirming her intrauterine pregnancy. Bedside ultrasound today does not show an intrauterine pregnancy. Her blood type is A negative and the father of the baby's blood type is not currently known. Which of the following is the best strategy for administration of Rh immunoglobulin?

She should receive Rh immunoglobulin 50 µg within 72 hours If a woman received Rh immunoglobulin in a previous pregnancy, is it required with subsequent pregnancies? Answer: Yes. Prevention: anti-D globulin at 28 weeks (and within 72 hrs of delivery if infant is Rh+)

A 23-year-old woman presents with vulvar itching and vaginal discharge. She is not currently sexually active and does not use contraceptives. She works in an office, drinks three alcoholic beverages a week, and recently took up long-distance running. Physical examination reveals a reddened vulva without surface ulcerations, the absence of lice or nits, normal-appearing vaginal secretions, and an unremarkable vaginal cavity. There is no adnexal tenderness. Which important item is missing from this patient's history?

Use of local irritants dx: vulvitis or vaginitis

A 31-year-old G1P0 woman who is 29 weeks pregnant presents to the hospital with contractions. She is found to be in acute preterm labor. Cervical exam reveals that she is 2 cm dilated. Which of following is the next appropriate step in management?

administer tocolytics in this woman in acute preterm labor.

A G2P2 46-year-old woman presents to her gynecologist's office with a six month history of irregular, heavy menses. She states her cramps are no worse than usual and she does not have dyspareunia or postcoital bleeding. On physical exam, she has a body mass index of 32 kg/m2. A bimanual examination reveals a normal-appearing cervix and a smooth, regularly-shaped uterus that is not enlarged. Which of the following is the most likely diagnosis?** A postmenopausal woman presents with abnormal vaginal bleeding. You suspect endometrial malignancy and refer the patient for further investigation. Which of the following modalities is initially used to determine a diagnosis? A 30-year-old woman presents to the office with dysmenorrhea, pelvic pain, and infertility for the last 5 years. What is the most appropriate next step to confirm a diagnosis of endometriosis? A 55-year-old postmenopausal woman presents to your office with a complaint of vaginal bleeding. Which of the following is the most appropriate next step in management? A pelvic exam reveals an immobile, retroverted uterus, palpable nodules to the posterior fornix, enlarged and tender left adnexa, and suprapubic tenderness. Which of the following conditions is this patient at increased risk for? A 32-year-old woman presents to the office for a wellness visit and to discuss trying to conceive. She reports heavy menstrual bleeding and chronic pelvic pain over the last 12 months. She describes the pain as dull, crampy abdominal pain that persists throughout her cycle and the majority of the month. She also describes dyspareunia and low back pain. She reports no alleviating factors and reports taking ibuprofen 400 mg regularly throughout her menstrual cycle. She reports no significant previous medical history and has never been pregnant. Vital signs include HR of 72 bpm, BP of 120/80 mm Hg, RR of 16/min, oxygen saturation of 99% on room air, and T of 98.7°F. Physical examination reveals vaginal tenderness and suprapubic tenderness. An immobile retroverted uterus is noted, with nodules in the posterior fornix. What is the most likely finding on surgical biopsy of the nodules? A 32-year-old woman with no significant medical history presents to her OB/GYN for evaluation of dyspareunia. She reports she has not had sexual activity for the past few months due to the significant amount of pain. She has tried condoms and lubricants without any improvement in her pain. Her last menstrual period was 2 weeks ago. Her vital signs are a HR of 78 bpm, RR of 16/min, SpO2 of 99% on room air, BP of 128/82 mm Hg, T of 97.8°F, and BMI of 27.1 kg/m2. On physical exam, the patient has significant pain when the speculum is opened, and an immobile uterus is noted on bimanual exam. Which of the following is the most appropriate management for this patient's condition?

endometriosis Endometrial hyperplasia Risk factors for endometrial cancer: nulliparity, obesity, unopposed estrogen (postmenopausal estrogen therapy without progestin), tamoxifen Pipelle endometrial suction curette Laparoscopy Endometrial biopsy Infertility Endometrial gland and stroma dx: Endometriosis Endometriosis Patient presents with pre- or mid-cycle dysmenorrhea, dyspareunia, dyschezia (painful bowel movement) PE may show uterosacral nodularity or a fixed or retroverted uterus or adnexal mass Definitive diagnosis is made by laparoscopy Most common site is ovaries Tx: NSAIDs, COCs, depot medroxyprogesterone acetate, GnRH agonists, surgery Can be caused by retrograde menstruation Oral contraceptives and ibuprofen

--------------------------------------------------------------

-----------------------------------------------------

According to the United States Preventive Services Task Force (USPSTF), which of the following is considered to put a pregnant woman at high risk for developing preeclampsia?

Autoimmune disease

What is the most common presenting symptom in a woman with vaginal cancer?

Vaginal bleeding Vaginal carcinoma accounts for only 3% of all gynecological cancers and is often asymptomatic. It is most commonly related to human papillomavirus (HPV) infection

A 51-year-old woman presents to the clinic complaining of irregular menstrual cycles as well as hot flashes, sleep disturbance, mood swings, and vaginal dryness. Which of the following would you expect to see on laboratory examination?

Elevated follicle-stimulating hormone dx: Perimenopause Menopause History of amenorrhea for 12 months Sx: hot flashes, sleep disturbances, depression, or vaginal dryness Labs: decreased estrogen and elevated follicle-stimulating hormone levels

A 14-year-old otherwise healthy boy presents to your clinic with his father. He is concerned because his breasts are enlarged. Appropriate management includes which of the following?

Reassurance dx: gynecomastia, a common finding in males during early pubertal changes A teenage male complains of a hard lump in his scrotum. Can this be monitored with reassurance? Answer: No! He must undergo further diagnostic testing with an ultrasound.

A 17-year-old girl presents with dysuria and vaginal itching. A speculum exam reveals the findings seen above. Which of the following statements is correct regarding this diagnosis?

The pH of the discharge is < 4.5 Candidal Vaginitis Risk factors: antibiotic use, diabetes Uncommon in prepubescent girls and postmenopausal women (not taking HRT) White cottage-cheese discharge Vaginal fluid pH 3.5-4.5 Pseudohyphae on saline microscopy Management:Fluconazole 150 mg PO single doseOTC invaginal azole therapy

Obstetrics consultation dx:Molar Pregnancy Patient presents with nausea, vomiting, abdominal pain, and vaginal bleeding PE will show uterine size that is larger than expected for dates Labs will show beta-hCG higher than expected for dates Diagnosis is made by ultrasound showing snowstorm or bag of grapes appearance Treatment is dilation and curettage New-onset hypertension < 20 weeks gestation: suspect molar pregnancy

An 18-year-old G1P0 woman at 16 weeks gestation presents with vaginal bleeding. She has had no prenatal care. Vital signs are unremarkable and physical examination only reveals a small amount of blood in the vaginal vault. A transvaginal ultrasound is performed as seen above. What is the next best step in management of this patient?

G2P1 woman at 13 weeks gestation presents for her initial prenatal visit. She has no ongoing medical conditions but previously underwent induction of labor due to preeclampsia with severe features. Today's vitals include HR of 91 bpm, RR of 18 breaths/min, BP of 129/87 mm Hg, T of 98.6°F, and SpO2 of 99% on room air. Exam reveals a palpable uterine fundus just superior to the pelvic rim with no other abnormal findings. Transvaginal ultrasound indicates a 13-week fetus with a fetal heart rate of 145 bpm. Which of the following is recommended to reduce the risk of preeclampsia in this pregnancy? A 25-year-old G1P0 woman at 34 weeks gestation presents to the clinic for a regular prenatal visit. She has noticed swelling in her feet but denies any associated pain as well as any other new symptoms. Temperature is 98.0°F, blood pressure is 155/92 mm Hg, and body mass index is 38.0 kg/m². Serum creatinine is 1.8 mg/dL. Which of the following is the most likely diagnosis?

Aspirin The most effective agent at preventing preeclampsia in moderate- to high-risk patient is low-dose aspirin, which should be started in the 12th or 13th week of gestation with continuation until delivery or shortly before delivery. Preeclampsia

A 27-year-old G2P1 woman at 38 weeks gestation presents to the clinic for a regular prenatal visit. She reports strong "kicks" in her lower abdomen but no other unusual symptoms. Vitals are normal. Physical exam reveals a soft mass in the lower abdomen. Ultrasound is ordered and reveals the fetal head in the fundus. Which of the following is the most likely diagnosis?

Breech presentation

A 13-year-old girl presents to the office with her mother, who is concerned she has not had a menstrual cycle. She is an avid runner, logging 30 miles per week. On exam, she exhibits no breast development or axillary or genital hair. Her mother was 15 when she started her menstrual cycle. What is your next step?

Begin workup for primary amenorrhea Amenorrhea Primary amenorrhea Failure of menses to occur by age 15 despite normal development of secondary sex characteristics Failure of menses to occur by age 13 in the absence of secondary sex characteristics Secondary amenorrhea Cessation of menses anytime after menarche has already occurred 3 months for those who have regular menses 6 months for those with irregular menses Lab workup may include FSH, LH, E2, prolactin, TFTs, testosterone, hCG Pregnancy is the most common cause

Which U.S. Food and Drug Administration pregnancy category is defined by the following: Animal reproduction studies have failed to demonstrate a risk to the fetus, and there are no adequate and well-controlled studies in pregnant women.

Category B

A 16-year-old girl presents to the pediatrician for a routine wellness examination. She reports that she has been sexually active since 15 years of age and in the past year has been monogamous with the same male sexual partner. The patient is up to date on vaccinations and had a negative HIV test at her appointment a year ago. The patient reports that she has no abnormal vaginal discharge, genital lesions, dysuria, or abdominal pain. Vital signs include a HR of 80 bpm, BP of 120/80 mm Hg, RR of 20/min, oxygen saturation of 98% on room air, and T of 98.6°F. Physical examination includes a regular rate and rhythm and a soft and nontender abdomen. The patient asks about what sexually transmitted infection screening is recommended. Which of the following is most appropriate?****?

Chlamydia trachomatis nucleic acid amplification test LOOK AT IMAGE

A 40-year-old G3P3A0 woman presents to the OB/GYN office for cervical cytology screening results. The results of her previous cervical cytology have been unremarkable. She has a history of diabetes mellitus type 2 and takes metformin 850 mg twice per day. Her vital signs are a BP of 131/81 mm Hg, HR of 81 bpm, RR of 16/min, SpO2 of 98% on room air, and a T of 98.7°F. Upon physical exam, normally developed genitalia are seen with no external lesions or eruptions. The vagina and cervix show no lesions, inflammation, or discharge. Her cervical cytology results show atypical glandular cells. What is the best next step in management?

Colposcopy with endometrial biopsy

A 26-year-old woman presents with a monochorionic, diamniotic twin pregnancy at 20 weeks of gestation in which polyhydramnios is visualized on ultrasound in twin one, and oligohydramnios in twin two. Doppler flow from placenta to both fetuses is good. The bladder of twin two is not visualized. Which of the following clinical interventions would be the best choice for this patient?******

Fetoscopic laser ablation of placental anastomoses dx: Twin-twin transfusion syndrome tx: Stage one twin-twin transfusion syndrome may be managed expectantly, The intervention of choice in Quintero stage two is fetoscopic laser ablation of placental anastomoses. This intervention utilizes a laser, inserted fetoscopically, to ablate the anastomotic vessels and halt the transfusion of blood from one twin to the other. Complications of the procedure include preterm premature rupture of membranes (PPROM), rupture of the inter-twin membranes, twin anemia polycythemia sequence, and intra-amniotic bleeding. stages Stage one refers to polyhydramnios/oligohydramnios, visualized bladders in both fetuses, and evidence of normal umbilical Doppler flow. Stage two is entered when the bladder of the donor fetus is not visualized on ultrasound. Further, if Doppler flow is abnormal in either fetus, stage three has begun. Stage four is when one or both of the fetuses show signs of hydrops, and stage five refers to the death of one or both fetuses

A 25-year-old sexually active woman presents to the emergency department with a complaint of painful vulvar ulcers and a swollen inguinal lymph node. She denies dysuria and vaginal discharge. Which of the following is most consistent with the description of the offending infectious organism?

Gram negative coccobacillus Haemophilus ducreyi dx: Chancroid Chancroid Risk factors: sexually active Sx: painful genital ulcers PE: papule evolves to a pustule which ulcerates, ulcers on an erythematous base covered by a gray or yellow purulent exudate and painful lymphadenopathy (bubo) Caused by Haemophilus ducreyi Tx: ceftriaxone 250 mg IM or azithromycin 1 g oral

A 27-year-old woman presents in the fall for her first prenatal visit. Her last menstrual period started 8 weeks ago. The patient reports she is in a monogamous relationship with one male partner. She has a family history of diabetes mellitus but has never been diagnosed with diabetes herself. Her body mass index is 21 kg/m2. Vital signs include a HR of 80 bpm, BP of 120/80 mm Hg, RR of 20/min, oxygen saturation of 98% on room air, and T of 98.6°F. Physical examination findings include a soft and nontender abdomen, no blood in vaginal vault, and a closed cervical os. Transvaginal ultrasound shows an intrauterine fetal pole with a fetal heart rate. Which of the following is indicated at this prenatal visit?

Inactivated influenza vaccine ------------------------------------------------- Group B Streptococcus screening (C) is recommended as part of routine prenatal care between 35 and 37 weeks gestation.

A 24-year-old patient presents to the emergency department and reports severe right-sided lower abdominal pain that began suddenly about 30 minutes ago and is associated with nausea. She has a temperature of 98.4°F, blood pressure of 152/72 mm Hg, and heart rate of 122 bpm. A serum beta-human chorionic gonadotropin is negative. On physical exam, the patient has moderate abdominal tenderness from palpation to the lower right quadrant, with active bowel sounds. A Doppler pelvic ultrasound reveals a 6 cm ovarian mass with impaired blood flow to the ovary. Which of the following structures does the patient's condition involve pathology in? A 15-year-old G1P0 girl at 23 weeks gestation presents with sharp, left lower quadrant abdominal pain for 1 hour. She previously had an ultrasound confirming the presence of a single intrauterine pregnancy. The pain is severe and associated with nausea. Pelvic examination reveals tenderness of the left adnexa. The patient's urinalysis is unremarkable. What test should be ordered to diagnose the patient?

Infundibulopelvic ligament dx: Ovarian torsion Ovarian Torsion Patient will be a woman, 15-30 years old or postmenopausal Sudden onset of unilateral (right > left) abdominal and pelvic pain Labs will show leukocytosis Imaging will show enlarged ovary or ovarian mass Definitive diagnosis and management: laparoscopy Pelvic ultrasound dx: ovarian torsion

A 19-year-old sexually active woman presents to your office with questions about cervical cancer screening. She wants to know when she should start getting screened since she's been sexually active for two years. Which of the following is the most appropriate next step in management?

Initiate screening at age 21 with cytology only HPV testing is not recommended in women under age 30 (A). Younger women are more likely to have transient HPV infections and testing at a young age leads to unnecessary colposcopies. Cervical cancer screening with cytology only (D) or co-testing with HPV (C) is not recommended in women under age 21 regardless of sexual activity.

A 28-year-old woman with a medical history of bipolar I disorder presents for medication counseling. She is actively trying to conceive and is currently taking valproate. She has no other significant medical history or current issues. Her vital signs are a BP of 122/73 mm Hg, HR of 67 bpm, SpO2 99% on room air, and T of 98.7°F. A physical exam is completed and is within normal limits. Which of the following medications is most appropriate to switch to at this time?******

Lamotrigine How long before conception should the patient change medication from valproate to lamotrigine? Answer: At least 3-6 months prior to conception. Black box= skin rash --------------------------------------------- Carbamazepine (A) is considered teratogenic, increasing the risk of congenital disorders such as spina bifida and cleft palate. Lithium (C) crosses the placenta and is associated with serious postnatal effects, including lithium toxicity causing low Apgar scores, bradycardia, feeding difficulties, or cardiac dysrhythmias. Quetiapine (D) is generally used to treat bipolar I disorder in patients who do not respond well to lamotrigine. Lamotrigine is first line because it has been studied more extensively and felt to be safe during pregnancy. Sertraline (E) is not indicated in treating bipolar I disorder and is more effective in treating major depressive disorder or anxiety disorders. This medication can be continued during pregnancy for those disorders after weighing the risks versus benefits.

A 32-year-old woman with no significant medical history presents to her OB/GYN for evaluation of dyspareunia. She reports she has not had sexual activity for the past few months due to the significant amount of pain. She has tried condoms and lubricants without any improvement in her pain. Her last menstrual period was 2 weeks ago. Her vital signs are a HR of 78 bpm, RR of 16/min, SpO2 of 99% on room air, BP of 128/82 mm Hg, T of 97.8°F, and BMI of 27.1 kg/m2. On physical exam, the patient has significant pain when the speculum is opened, and an immobile uterus is noted on bimanual exam. Which of the following is the most appropriate management for this patient's condition?

Oral contraceptives and ibuprofen endometriosis Patient presents with pre- or mid-cycle dysmenorrhea, dyspareunia, dyschezia (painful bowel movement) PE may show uterosacral nodularity or a fixed or retroverted uterus or adnexal mass Definitive diagnosis is made by laparoscopy Most common site is ovaries Tx: NSAIDs, COCs, depot medroxyprogesterone acetate, GnRH agonists, surgery

A male child born at 40 weeks of gestation has an Apgar score of 4 at birth. He is gasping for air and has a pulse of 68 despite positioning and airway clearance. Which of the following would be the best choice for initial clinical intervention in this patient?

Positive pressure ventilation A neonate who demonstrates a weak cry with gasping and a pulse less than 100 beats per minute should receive positive airway pressure within one minute of birth. APGAR Appearance, pulse, grimace, activity, respiration Calculated at 1 and 5 minutes after birth Score of 0, 1, or 2 per section Baby with low score may require intervention

3-year-old girl presents to the ED with her parent and grandparent for evaluation of vaginal spotting. The child has no other concerns and specifically does not report sexual abuse or trauma when questioned without family. After watching a news special on sexual abuse, the parent is concerned that her daughter may have been sexually assaulted, despite no specific reports. The child is acting appropriately, and there are no external lesions or signs of trauma. The internal pelvic exam is difficult due to the patient's age. Which of the following statements applies to this patient?

She may have a vaginal foreign body

A 40-year-old woman with a history of asthma presents to the ED with symptoms of wheezing and shortness of breath similar to previous exacerbations. Her vital signs are BP 115/70 mm Hg, HR 80 bpm, RR 14/min, and pulse oximetry 99% on room air. The patient is offered and agrees to a point-of-care beta-hCG test that returns positive. Upon further questioning, patient denies any vaginal or urinary complaints. On exam, you note mild bilateral wheezing with good air movement. Which of the following is the most appropriate next step in management?

Treat her asthma as indicated and, if improved, discharge with outpatient obstetrical follow-up

1)The United States Preventive Services Task Force recommends screening mammography for women every two years from ages 50-74 years. 2)Which of the following is the most effective recommendation? Etonogestrol implant or Intrauterine devices (IUDs) 3)Staphylococcus aureus is the most common causative pathogen of a breast abscess i 4)You decide to perform a potassium hydroxide wet preparation of a sample. Which of the following secretion qualities does this test evaluate?Odor 5)Which of the following lifestyle changes commonly leads to secondary amenorrhea? Strenuous exercise 6)Screening for gestational diabetes should be performed during which of the following weeks of gestation? 24-28 weeks 7)recommended to use as an anti-hypertensive in a pregnant woman? Alpha-methyldopa 10)Which of the following is a risk factor for occiput posterior position at birth? nulliparous, greater risk of lower Apgar scores, umbilical artery acidemia, and neonatal intensive care admission. 11)What is the most common nonviral sexually transmitted disease in the world? Answer: Trichomonas vaginalis. 12)What is the most frequent cystic structure found in the ovary?Follicular cyst 13)What is the most common type of gynecological cancer? Answer: Uterine cancer. 14)Chlamydia trachomatis = Azithromycin may be given in a one-time oral dose of 1 gram in pregnant patients. Doxycycline is given to nonpregnant patients 100 mg twice per day for seven days and should be avoided in pregnant individuals. 15)Which of the following test results indicated the patient's need for Rh(D) immunoglobulin? Negative Rh(D) on blood typing and antibody screen 16) Which of the following factors increases a woman's risk for uterine prolapse? = Increasing parity, vaginal delivery, advanced age, hysterectomy and obesity. 17)What are common side effects of danazol? Answer: Androgenic side effects such as decreased breast size, weight gain, acne, and hirsutism. 18)A gestational sac with a yolk sac in the uterus on transvaginal ultrasound is diagnostic of an IUP. 19)A new first-time mother calls for advice on nipple pain with breastfeeding. She is 6 days post partum after an uncomplicated delivery. Which one of the following is most effective for reducing the pain? Education on positioning 20)Which of the following laboratory studies should be performed on all pregnant women during the initial prenatal visit? Urine culture 21)Which of the following is the best method to estimate the delivery date in this patient? Fetal crown-rump length due date Naegele Rule 1st day of last menstrual period (LMP) + 7 days - 3 months + 1 year 22)Which of the following medications is contraindicated in pregnancy? Captopril 23)Infertility is defined as failure of conception after how long trying to conceive? Answer: One year. 24) You are treating a patient for primary amenorrhea. Her diagnostic evaluation has determined that the etiology is primary ovarian failure. Which of the following disorders is this patient at risk for? Osteoporosis 25)What is the definition of preterm or premature labor? Answer: Labor before 37 weeks. A delivery before 20 weeks is considered a miscarriage. 26)A 19-year-old G1P0 woman at 26 weeks presents with abdominal pain after being involved in a motor vehicle collision. External pelvic examination reveals vaginal bleeding. Which of the following is true regarding this presentation? Emergent fetal monitoring and obstetric consultation are required Which of the following laboratory abnormalities is consistent with the most likely diagnosis? Hypofibrinogenemia 27)Costovertebral angle tenderness (B) indicates upper urinary tract infection such as pyelonephritis. 28)What is the management of PID in a pregnant patient? Answer: Hospitalization and second generation cephalosporin (IV cefoxitin or cefotetan) and 1 gram azithromycin PO. 29)Surgical removal is associated with a 25% to 30% rate of recurrence = uterine fibroids Uterine fibroids decrease in size during menopause (A) and enlarge early in pregnancy. Uterine fibroids are two times more common in African American women (B) than in Caucasian women. Uterine fibroids are usually multiple in nature rather than single (D). 30) Which of the following pharmaceutical agents is the most commonly used to induce labor in a woman with a favorable cervix? Oxytocin 31)A woman presents with lower abdominal pain for five days. You consider pelvic inflammatory disease as a diagnosis. Which of the following increases the risk of this condition? Multiple sexual partners 32) A 27-year-old woman is on an estrogen-progestin oral contraceptive. Which of the following mechanisms of action is the most important for providing contraception? Inhibition of the midcycle luteinizing hormone surge 33)SBP ≥ 140 mm Hg or DBP ≥ 90 mm Hg for the first time during pregnancy at ≥ 20 weeks gestation but no associated proteinuria is present = Gestational hypertension

facts

What is the surgical intervention method of choice for a premenopausal woman with a leiomyoma who wishes to retain her fertility? What is the diagnostic test of choice in a patient with a suspected leiomyoma?

Myomectomy Transabdominal and transvaginal ultrasound

A 25-year-old G2P1 woman presents to your office at 32 weeks gestation with a complaint of severe itching, particularly on the palms of her hands and the soles of her feet. Lab results reveal elevated bile acids. Regarding this disease, which of the following statements is most correct?***

There is an increased risk for fetal demise dx: Intrahepatic cholestasis of pregnancy. tx: Ursodeoxycholic acid

A 60-year-old woman seen in the clinic presents with a palpable breast mass, which is concerning for breast cancer. She had a negative screening mammogram six months ago. Which of the following is the first step in the diagnostic workup?

Diagnostic mammogram and ultrasound

Which of the following correctly describes physiologic changes that occur in pregnancy?

Blood volume increases Cardiac output, blood volume and heart rate increase in pregnancy. The uterine fundus grows outside the pelvis into the abdomen at 12 weeks gestation

A 17-year-old G1P0 adolescent at 25-weeks gestation presents with intermittent blurred vision. On presentation, she is currently asymptomatic. Vital signs are HR 84, BP 165/97, oxygen saturation 97%. Physical examination reveals 2+ pitting edema on both lower extremities and urinalysis has 3+ protein on dip. Which of the following is the next best step in management?

Admit for further obstetric evaluation dx: severe preeclampsia Treatment: delivery at 37 weeks (without severe features) and 34 weeks (with severe features) AND prevention of seizures with magnesium sulfate and prevention of permanent maternal organ damage New-onset hypertension < 20 weeks gestation: suspect molar pregnancy

A 45-year-old woman presents with heavy menstrual bleeding for the past several months. Her cycles are every 30 days and are regular. Each period lasts about 7 days and is the heaviest during the first 5 days, requiring a tampon change once every hour. Her periods are getting more painful, and the pain is not relieved with ibuprofen. Her temperature is 98.5°F, blood pressure is 119/78 mm Hg, heart rate is 98 bpm, and respiratory rate is 22/minute. A bimanual exam reveals a tender, soft, mobile, and uniformly enlarged uterus. Her hemoglobin is 10.1 g/dL, and a pregnancy test is negative. Which of the following is the most likely diagnosis?

Adenomyosis endometrial glands and stroma are ectopically present within the myometrium (uterine musculature) abnormal uterine bleeding, dysmenorrhea, chronic pelvic pain, and uterine enlargement. Adenomyosis may be diffuse or focal. Diffuse adenomyosis is characterized by a uterus that is uniformly enlarged, soft, mobile, and tender to palpation o Adenomyosis is a largely clinical diagnosis definitively diagnosed is by a histologic exam of surgical specimens procured during hysterectomy. Ultrasonography is the preferred imaging tool of choice, An MRI finding of a junctional zone (area located between the endometrium and myometrium) thickness ≥ 12 mm is considered diagnostic for adenomyosis Nonsteroidal anti-inflammatory drugs can be used in cases of mild to moderate pain Hormonal therapie Hysterectomy remains the definitive treatment of this condition.

n 18-year-old woman at 37 weeks gestation presents with a spontaneous leakage of fluid from the vagina. She has no other signs of active labor. Vital signs are unremarkable and the patient has no complaints except for the leakage of fluid. What management is indicated? A G1P0 woman who is 38 weeks pregnant presents to the clinic reporting a sudden gush of clear fluid from her vagina. The leakage was so significant that it soaked her clothes and bed sheets. She denies contractions. Which of the following would be expected finding in this patient?

Admit to obstetrics for delivery Pooling of amniotic fluid in the vaginal fornix on physical exam dx: prelabor rupture of membranes (PROM) at 37 weeks of gestation Nitrazine test: Amniotic fluid turns nitrazine paper from orange to dark blue (+) Ferning = Amniotic fluid Amniotic fluid usually has a pH range of 7.0 to 7.3 compared to the normally acidic vaginal pH of 3.8 to 4.2. Prelabor Rupture of Membranes (PROM) Membrane rupture prior to labor Preterm PROM (PPROM): PROM occurring at < 37 weeks Fluid ferning + blue nitrazine paper = amniotic fluid Admission, OB consultation

The presence of which one of the following best differentiates a diagnosis of premenstrual dysphoric disorder from premenstrual syndrome?

Affective symptoms Premenstrual Dysphoric Disorder (PMDD) Five or more symptoms (eg. mood swings, irritability, change in appetite or sleep, breast tenderness, bloating, weight gain) present during the week prior to menses, resolving within a few days after menses starts Symptoms present for most of the preceding year Patients who require treatment have symptoms associated with significant distress and interference with work, school, and relationships Management includes lifestyle modifications and pharmacologic options (SSRIs are first line) Key point between premenstrual syndrome and premenstrual dysphoric disorder is PMDD symptoms usually hinder personal or professional life (unlike premenstrual disorder)

A 25-year-old G1P0 woman at 33 weeks gestation presents to her obstetrician with a complaint of wetness and leakage from her vaginal area. On physical exam, fluid can be seen coming from the cervical canal. The cervix is not dilated. Fern test is positive. Which of the following medications should be given at this time to delay delivery? A 22-year-old woman at 36 weeks gestation presents with complaints of feeling a sudden gush of water coming from her vagina. She has had regular obstetrical follow-up and a normally progressing pregnancy. Which of the following is the most appropriate next step in management?

Ampicillin dx: Preterm PROM "When membrane rupture occurs before 37 weeks gestation" Microscopic evaluation of vaginal fluid

Which of the following contributes to a higher Bishop score? AAnterior cervical position BCervical effacement of 0 to 30% CFetal station of -3 DFirm cervical consistency

Anterior cervical position score of the likelihood of vaginal delivery following induction b

28-year-old G3P0 woman presents to the office to discuss recurrent pregnancy loss and recent amenorrhea. She has not had a menstrual cycle for the last 2 months. She reports previously having a regular menstrual cycle of 28 days. Age of menarche was 14. Previous medical history is significant for anxiety, controlled well with sertraline. Surgical history is significant for three dilation and curettage procedures for retained products of conception, most recently 3 months ago. Vital signs include HR of 60 bpm, BP of 120/80 mm Hg, RR of 18/min, oxygen saturation of 99% on room air, and T of 98.7°F. Physical examination reveals a well-nourished woman, and findings on the exam are within normal limits. What is the most likely diagnosis?

Asherman syndrome Intrauterine adhesions appear to result from trauma to the basalis layer of the endometrium. Intrauterine manipulation is associated with the development of adhesions The main components of the diagnostic evaluation for intrauterine adhesions are medical history and uterine cavity evaluation. Hysteroscopy allows for simultaneous diagnosis and treatment.

A 32-year-old G4P3 woman with a history significant for polyhydramnios is brought to the emergency department 15 minutes after a home birth. She complains of persistent vaginal bleeding with thick clots. She is anxious and diaphoretic. Her physical exam is notable for a large, boggy uterus. She receives oxytocin intramuscularly as intravenous access is started. Repeat examination reveals a persistently boggy uterus with significant active vaginal bleeding. Which of the following is an absolute contraindication to the use of carboprost in the management of this patient? ================================================================ A 30-year-old woman who had an uncomplicated pregnancy delivered a healthy newborn a few minutes ago. She just now delivered a portion of the placenta with large areas absent of cotyledons. Copious amounts of blood begin to flow from the vaginal orifice. Within minutes, the patient becomes lightheaded but maintains consciousness. You attempt manual extraction but obtain minimal pieces of placental tissue. The bleeding continues, and the blood pressure decreases to 78/68 mm Hg while heart rate increases to 136 bpm. Which of the following is the best next step in management of postpartum hemorrhage? ================================================================= A 26-year-old woman just vaginally delivered twin, macrosomic boys. This was her fourth pregnancy, which has been the longest of them all at 43 weeks gestation. Her delivery was difficult and required forceps to complete. Estimated blood loss was 1,100 mL. Currently, she is hypotensive, tachycardic, and anemic. Which of the following is the most likely diagnosis? ============================================================= A 35-year-old obese woman who just delivered a 10 lb baby after 48 hours of labor starts hemorrhaging from the vaginal area and has a sudden drop in blood pressure. She was in her 41st week of gestation when she was admitted to have labor induced. Which of the following is the best initial step in diagnosing the underlying cause of bleeding?

Asthma dx; Postpartum hemorrhage is an obstetrical emergency First line treatment of uterine atony is manual uterine massage and uterotonic agents. Oxytocin is typically the first agent used, with carboprost and methylergonovine typically used in cases of continued bleeding after oxytocin use. Carboprost, a prostaglandin analog that stimulates uterine contractility, may cause significant bronchospasm and is contraindicated in patients with asthma. -=================================================== Transfer to the operating room dx: Postpartum hemorrhage ==================================================== Uterine atony postpartum hemorrhage n atonic uterus feels soft and boggy. Treatment includes uterine massage and intramuscular methylergonovine maleate or oxytocin ============================================= Palpate to determine the size and firmness of the uterus dx: Postpartum hemorrhage

A 28-year old woman presents with several painful ulcers she has developed in the vaginal area. Examination reveals multiple 0.5 cm to 1.5 cm oval ulcers with sharply defined borders and a yellowish-white membrane. She denies recent sexual activity. Except for recurring aphthous ulcers of her mouth, her past history is unremarkable. Which of the following is the most likely diagnosis?

Behçet syndrome Behçet syndrome included recurring genital and oral ulcerations and relapsing uveitis. The genital and oral ulcers are classically painful with a necrotic center and surrounding red rim.

A 21-year-old woman with no prenatal care presents for evaluation of lower abdominal pain and fever. She estimates that she is approximately 7.5 months pregnant. On questioning, she acknowledges intermittent pain for two days and a gush of fluid shortly after the pain began. Her temperature is 101.8°F. Physical examination is notable for purulent material in the vaginal vault. Which of the following is the most likely diagnosis?

Chorioamnionitis (intra-amniotic infection) Infection, inflammation, or both of the amniotic fluid, placenta, fetus, fetal membranes, or decidua Risk factors: nulliparity, prolonged rupture of membranes, meconium-stained amniotic fluid, internal fetal or uterine contraction monitoring Genital tract infection: STIs, group B Streptococcus, bacterial vaginosis Rx: ampicillin + gentamicin

A young woman has a significant family history of gynecologic malignancy. She refuses oophorectomy at this time. Which of the following serum levels will you monitor, in addition to serial ultrasound screening? Which of the following is the most common cell type of ovarian cancer?

Cancer antigen 125 dx: ovarian cancer Epithelial

A 23-year-old, sexually active woman presents with abdominal pain. Vital signs are normal. Pelvic examination reveals cervical motion tenderness and bilateral adnexal tenderness. Which of the following treatments is most likely indicated? A 22-year-old woman presents to your office with complaints of pelvic pain and vaginal discharge. She reports having recent unprotected intercourse with a new partner and her last menstrual period ended three days ago. Which of the following is the most appropriate pharmacologic treatment?

Ceftriaxone 500 mg IM once + doxycycline 100 mg PO BID for 14 days + metronidazole for 14 days dx:pelvic inflammatory disease (PID) Pelvic Inflammatory Disease (PID) History of multiple sexual partners or unprotected intercourse Lower abdominal pain, cervical motion tenderness, painful sexual intercourse PE will show mucopurulent cervical discharge Most commonly caused by Chlamydia trachomatis Outpatient treatment is ceftriaxone + doxycycline + metronidazole Fitz-Hugh-Curtis syndrome: perihepatitis + PID What is the management of PID in a pregnant patient? Answer: Hospitalization and second generation cephalosporin (IV cefoxitin or cefotetan) and 1 gram azithromycin PO.

A 24-year-old woman presents for follow-up after an abnormal Pap smear. Colposcopy is performed and reveals a white, sharply demarcated area on the cervix after application of acetic acid. What is the most likely diagnosis?

Cervical dysplasia Schiller test using Lugol's solution

A 38-year old woman presents with right upper quadrant pain that is worse with deep breathing. She reports having multiple sexual partners. She was recently treated for "an STD," however, she didn't complete the entire course of antibiotics because "she felt fine." What is the most likely organism causing this condition?

Chlamydia trachomatis dx:Fitz-Hugh-Curtis syndrome or perihepatitis

Which of the following represents a risk factor for developing vulvar cancer? A 73-year-old postmenopausal woman with a past medical history significant for genital warts presents to the outpatient gynecology office complaining of a solitary, pruritic plaque on the right labia majora. She first noticed it six months ago but thought it was a result of new body soap. Discontinuation of the soap and hydrocortisone cream did not relieve symptoms. She denies discharge, abdominal pain, hematuria or constitutional symptoms. What is the most likely diagnosis? A 62-year-old woman with a history of diabetes mellitus type 2 presents to her primary care office with vaginal itching. She reports noticing itching of her external genitalia, which has gradually increased over the past 6 months. The patient states there are no alleviating or aggravating factors, and she has not been using any new soaps, detergents, or hair removal products. She reports using over-the-counter emollients with no relief. The patient takes metformin 500 mg twice daily and has a 30 pack-year smoking history. Vital signs are a BP of 134/82 mm Hg, HR of 81 bpm, RR of 16/min, SpO2 of 98% on room air, and a T of 98.7°F. Physical exam reveals one 3 × 2 cm white lesion on the left labia majora. The lesion does not bleed when manipulated. What is the best next step in management?

Cigarette smoking Vulvar cancer history of human papillomavirus (HPV) infection and clinical presentation. Vulvar cancer is most common in postmenopausal women. -single pruritic lesion located on the labia majora. HPV, cigarette smoking, northern European ancestry, history of vulvar dystrophy, immunocompromised state, prior cervical cancer, and vulvar or cervical intraepithelial neoplasia are risk factors for developing vulvar cancer. Suspicious lesions should be biopsied. Vulvar Cancer History of human papillomavirus (types 16, 18, 33) Vulvar lesion and pruritus PE will show unifocal vulvar ulcer, plaque, or mass, predominantly on the labia majora Most common type is squamous cell carcinoma (SCC) Which type of vulvar cancer is often misdiagnosed as an abscess or cyst? Answer: Bartholin gland carcinoma.

A 24-year-old woman presents to the clinic for follow-up on her cytology results from her Pap test. Vital signs include a HR of 80 bpm, BP of 120/80 mm Hg, RR of 20/min, oxygen saturation of 98% on room air, and T of 98.6°F. Physical examination findings include a regular rate and rhythm and a soft and nontender abdomen. You discuss with the patient that her Pap test cytology showed high-grade squamous intraepithelial lesion. She did not have human papillomavirus testing obtained. Which of the following is the most appropriate next step in management?

Colposcopy

A 19-year-old woman presents to the clinic with heavy and painful periods for the past year. Her cycles are irregular lasting up to three to four weeks at a time. The patient is not on any medications, is sexually active, and a pregnancy test in the office is negative. She has a normal body mass index and Pap smear, and the pelvic ultrasound and pelvic examination are unremarkable. Which of the following is the first-line treatment for this condition? A 46-year-old woman presents with heavy menstrual bleeding for the past three menstrual cycles. Her bleeding is heavier than normal and her menstrual cycle lasts longer than what has been typical for her. She denies any weight changes or cold intolerance. Labs demonstrate a mild microcytic anemia, but are otherwise within normal limits. A pelvic ultrasound is ordered and shows no abnormalities. An endometrial biopsy showed secretory endometrium and is negative for any malignancy or other abnormalities. What is the most likely diagnosis? A 32-year-old woman presents with vaginal bleeding for two weeks. She states she has had to change her pad every 2-3 hours with the bleeding. Vital signs are normal and physical exam only reveals blood coming from the cervical os. The patient's hemoglobin is 12 g/dL and her pregnancy test is negative. What treatment is indicated for this patient?****

Combined oral contraceptive pills dx: Abnormal uterine bleeding (AUB)

A 32-year-old woman presents with vaginal bleeding for two weeks. She states she has had to change her pad every 2-3 hours with the bleeding. Vital signs are normal and physical exam only reveals blood coming from the cervical os. The patient's hemoglobin is 12 g/dL and her pregnancy test is negative. What treatment is indicated for this patient? A 30-year-old woman presents with abnormal menstrual bleeding. Her history and physical are unrevealing. Which of the following is the best next step, other than laboratory testing, in evaluating this patient?

Combined oral contraceptive pills dx: Abnormal uterine bleeding (AUB) Stable patients with mild anemia should also be treated with oral iron supplementation. Pelvic ultrasonography

A 27-year-old G3P2 woman goes into labor at 39 weeks gestation. Her vital signs are a BP of 117/65 mm Hg, HR of 78 bpm, SpO2 of 97% on room air, and T of 98.7°F. A speculum exam confirms rupture of membranes. The cervix is dilated to 4 cm and there is 50% effacement. An ultrasound confirms cephalic fetal presentation. External monitoring is placed and fetal heart tracing shows a baseline rate of 145 bpm. Continued monitoring demonstrates moderate variability without late or variable decelerations. Some accelerations are noted as well. What is the best next step in managing this patient's care?

Continue expectant management baseline heart rate of 110-160 bpm, Fetal Heart Rate: Mnemonic VEAL CHOP Variable: cord compression Early: head compression Accelerations: okay Late: placental insufficiency ------------------------------------------------ Beginning amnioinfusion (A) is an option when there are recurrent significant variable decelerations indicating significant cord compression and hypoxemia. Performing cesarean delivery (C) would be necessary if the decelerations are classified as late or if there are significant variable decelerations that persist despite conservative measures to alleviate cord compression. Placing fetal scalp electrodes (D) is helpful in monitoring variable decelerations when there is a fetal heart rate that is nonreassuring. Placing a patient in the lateral Sims position (E) is an option when there are recurrent significant decelerations indicating cord compression. Changing the maternal position is sometimes helpful to alleviate cord compression.

A 34-year-old woman presents with concerns about contraception. She is in a monogamous relationship and wishes to maintain long-term fertility. She has regular menstrual cycles that are typically 28 days and consist of light bleeding for 4-5 days. She experienced menarche at age 11. Her medical history is significant for essential hypertension and hypothyroidism. She is currently taking lisinopril and levothyroxine. Her vital signs are a BP of 145/96 mm Hg, HR of 88 bpm, SpO2 98% on room air, and T of 98.4°F. A physical examination is completed and no abnormalities are noted. She is concerned about taking a prescription that might cause weight gain or nausea. Which of the following is the most appropriate contraceptive method for this patient?

Copper-containing intrauterine devic

An 18-year-old G1P0A1 woman presents to the OB/GYN office with concern for an unplanned pregnancy. The patient reports having unprotected intercourse with her partner last night. She states she is not planning on becoming pregnant until she is in her late 20s. Vital signs are a BP of 123/78 mm Hg, HR of 86 bpm, RR of 16/min, SpO2 of 98% on room air, and a T of 98.7°F. A human urine chorionic gonadotropin result is negative. On physical exam, normally developed genitalia are noted with no external lesions or eruptions. The vagina and cervix show no lesions, inflammation, discharge, or tenderness. What is the best next step in management?

Copper-containing intrauterine device Emergency contraception

A G1P0 woman at 39 weeks 5 days gestation presents to the labor and delivery ward and reports spontaneous rupture of membranes 30 minutes prior to her arrival at the hospital. Initial examination reveals 4 cm of dilation and 70% effacement of the cervix, with repeat examination in 2 hours demonstrating 6 cm of dilation and 85% effacement. The patient does not progress past 6 cm after an additional 5 hours. Which of the following is the most appropriate management of labor at this time?

Deliver via cesarean dx:Labor arrest Arrest in the active phase is classified in a patient with ≥ 6 cm of cervical dilation with membrane rupture and either no cervical change for ≥ 4 hours despite adequate contractions or no cervical change for ≥ 6 hours with inadequate contractions.

When educating a patient about risks of undergoing surgical repair of a rectocele, which of the following potential complications should be mentioned?**** Which of the following represents a common risk factor for the development of a rectocele?

Dyspareunia Obesity most effective nonsurgical option is the use of a pessary. Medications to address constipation should also be used if this is a clinical feature of the patient's disorder. The most common surgical management strategy is a posterior colporrhaphy which has an anatomic cure rate of up to 96%.

A 25-year-old woman presents to her obstetrician for evaluation of recurrent second trimester pregnancy loss. During each of her previous two pregnancies, she presented during week 15 with painless cervical dilation and effacement with subsequent rapid delivery of a preterm infant. Which of the following is a risk factor for this woman's cervical insufficiency? She is currently twelve weeks pregnant. Which of the following is the most appropriate next step in management in this patient?

Ehlers-Danlos syndrome dx: Cervical insufficiency= inability of the uterine cervix to retain a pregnancy throughout the second trimester, in the absence of uterine contractions. Cervical cerclage

A 26-year-old healthy woman presents to the emergency department with abdominal pain. She was at home when she developed sudden onset lower abdominal pain followed by a brief syncopal episode. Her vital signs include blood pressure of 88/46 mm Hg, heart rate of 112 bpm, respiratory rate of 18/min, temperature of 37.6°C, and oxygen saturation of 98%. She had a positive home pregnancy test yesterday. After initiating aggressive resuscitation, what is the most appropriate next step in management? ***? Which of the following is the greatest risk factor for an ectopic pregnancy? A 23-year-old woman presents with pain in the suprapubic area and right lower quadrant of her abdomen that began 4 hours ago. She denies vaginal discharge and vaginal bleeding. Her vital signs are BP 115/65, HR 60, RR 12, and T 98.1°F. Her last menses was one week ago. Which of the following diagnostic tests should be obtained first? A 26-year-old woman presents with abdominal cramping after a positive home pregnancy test. She states that the pregnancy is wanted. Her vitals are T 98.7°F, HR 94 bpm, BP 110/66 mm Hg, RR 18/min, and oxygen saturation 97%. Her exam is unremarkable. Labs reveal a serum beta-hCG of 1,000 mIU, and she is Rh-positive. An ultrasound is performed as seen above. Which of the following is the appropriate management for this patient? An 18-year-old woman comes to the emergency department with sudden onset of right lower abdominal pain. She describes the pain as crampy and intermittent. Her last menstrual period was 7 weeks ago. She has noticed vaginal spotting during the past few days. Physical examination shows a tender lower abdomen with guarding. Pelvic examination shows a slightly enlarged uterus, cervical motion tenderness, and a palpable adnexal mass. Which of the following is the most likely diagnosis?

Emergent OB/GYN consult dx: Ectopic pregnancy Previous ectopic pregnancy Urine beta-hCG Discharge home with repeat beta-hCG in 48 hours [look at image]]] Her transvaginal ultrasound shows an early gestational sac without a yolk sac or fetal pole within the uterus. Labs will show positive pregnancy test and lower than expected serum beta-hCG levels The definitive ultrasound finding for an intrauterine pregnancy (IUP) would be the presence of a yolk sac or fetal pole. Most commonly located in a fallopian tube Which of the following confirms an intrauterine pregnancy?Intrauterine fetal pole and yolk sac Ectopic pregnancy

55-year-old woman presents to the oncology clinic to discuss therapy options for her newly diagnosed, non-metastatic, unilateral, invasive breast cancer. Routine analysis reveals estrogen receptor-positive and progesterone receptor-positive tumor expression. Which of the following is the recommended therapy for this patient?

Endocrine therapy with breast surgery Tamoxifen, an estrogen receptor modulator, significantly reduces the risk of recurrence and death in patients with ER-positive disease, such as the patient in the vignette.

A 41-year-old woman suffers from heavy and irregular menses, which at times leads to fatigue, lightheadedness, and dyspnea. She has had three hospitalizations in the past year for such episodes. Her gynecologic evaluation has not revealed any pathological cause. The heavy menses continue despite hormonal therapy. Which of the following treatment options should be considered next?

Endometrial ablation dx: Abnormal uterine bleeding --------------------------------------------------- Colposcopy (A) is a gynecologic procedure in which the cervix, vagina, and vulva are directly visualized under illumination and magnification. Hysteroscopy (D) is the current gold standard for evaluating uterine pathology. Both of these diagnostic tests would neither treat nor prevent further menstrual bleeding. Hysterectomy (C) may be a last resort, but endometrial ablation is usually attempted first, as hysterectomy has a high risk of perioperative complications and prolonged recovery.

A 16-year-old girl is seen in clinic due to a concern for a breast mass that she notes has been present for 3 months. The size of the mass has not changed over the previous 3-months. She is otherwise healthy and is not on any medications. On exam you note a 2 cm diameter mass on the right outer quadrant of her right breast that is rubbery, mobile, and not tender. Which of the following is the most likely diagnosis?

Fibroadenoma Patient will be a woman of childbearing age PE: painless, firm, solitary, mobile, slowly growing breast mass Treatment: conservative management or surgical excision Most common breast tumor in adolescents

A 32-year-old woman with fluctuating menstrual intervals comes to your primary care clinic for an evaluation. She is not currently pregnant but has had two normal pregnancies with healthy deliveries. On her pelvic examination during adnexal palpation, you notice that one ovary is appreciably larger than the other. In your discussion with the patient after the exam, you explain that this ovarian enlargement most likely represents which one of the following conditions?

Functional ovarian cyst

A 14-year-old girl with onset of menarche 6 months prior reports irregular menstrual cycles, with variations between cycle lengths of as much as 12 days. Most of her periods consist of painless, heavy bleeding. She is currently experiencing such a period, with blood loss that requires frequent changing of menstrual pads with use of 10 or more pads per day. She has no history of epistaxis or easy bruising and takes no medications. Bimanual and speculum exam reveals copious blood in the vaginal vault with no identifiable nonuterine source. The uterus is of normal size and contour without tenderness. Adnexa are nontender to palpation without identifiable masses or abnormalities. She has been sexually active with one partner and her urine pregnancy test result is negative. What is the most likely cause for this patient's menstrual irregularities?

Hypothalamic-pituitary-ovarian axis immaturity

A 36-year-old G1P0 woman presents at 32 weeks gestation with right upper quadrant abdominal pain. She has no past medical history and her pregnancy has thus far been uncomplicated. Her vital signs on arrival are T 37.3°C, HR 110 bpm, BP 125/75 mm Hg, RR 24/min. Her physical exam is significant for moderate right upper quadrant tenderness to palpation. Her laboratory studies are remarkable for WBC 14 x 109/L, hemoglobin 9 g/dL, hematocrit 27%, platelets 70 X 109/L, AST 120 U/L, ALT 100 U/L, total bilirubin 1.5 mg/dL and LDH 1,000 U/L. Which of the following is the most likely diagnosis? Which of the following combination of laboratory findings is most suggestive of hemolysis, elevated liver enzymes, and low platelet count syndrome of pregnancy?

HELLP syndrome Sx: RUQ abdominal pain, nausea Labs:microangiopathic hemolytic anemia (low hemoglobin and schistocytes on blood smear )low platelets (<100,000 cells/microL) elevated liver enzymes (≥2x upper limit of normal) Management is blood pressure management, magnesium sulfate for prevention of eclamptic seizures, and delivery of the fetus When is HELLP syndrome most commonly diagnosed? Answer: 28-36 weeks gestation. Schistocytes, thrombocytopenia, and elevated aspartate aminotransferase and alanine aminotransferase greater than twice normal

A healthy 35-year-old G0P0 woman presents to the office as a new patient. She states she has not had routine gynecologic care for the past 10 years. She reports 2 months of heavy vaginal bleeding during menses and postcoital spotting. She describes associated symptoms of malodorous, white vaginal discharge occurring daily and frequent low back pain. She states she has never experienced anything like this before. She reports no significant medical history. She was 12 years of age at menarche. Sexual history is significant for multiple partners over the last 16 years. She currently reports a monogamous sexual relationship with a man. Vital signs include HR of 72 bpm, BP of 120/80 mm Hg, RR of 20/min, oxygen saturation of 97% on room air, and T of 98.7°F. Physical examination reveals a well-nourished woman. Lymphadenopathy is present in the right groin. Pelvic examination reveals a 4 cm exophytic lesion on the right side of the cervix, creating a barrel-shaped cervix. A scant amount of watery, purulent discharge is noted in the vaginal vault. The uterus is enlarged and boggy on bimanual examination. There is no cervical motion tenderness. Which of the following pathogens is most likely responsible for the presumed diagnosis?

Human papillomavirus [HPV 16 and 18] dx: Cervical cancer

A 50-year-old G2P2A0 woman presents to the OB/GYN office for follow-up regarding her cervical cytology result. Her history is significant for hypertension, and she takes lisinopril 10 mg daily. She has been coming in for her routine Papanicolaou smear every 3 years with no abnormal results in the past. On her most recent cervical cytology, atypical squamous cells of unknown significance were found. Vital signs are a BP of 127/78 mm Hg, HR of 86 bpm, RR of 16/min, SpO2 of 98% on room air, and a T of 98.7°F. On pelvic exam, normally developed genitalia are noted with no external lesions or eruptions. The vagina and cervix show no lesions, inflammation, discharge, or tenderness. What is the best next step in evaluation of the patient's cytology results?

Human papillomavirus testing atypical squamous cells of undetermined What is the most common abnormal finding on cervical cytology? Answer: Atypical squamous cells of undetermined significance.

A 23-year-old G1P0 woman with an estimated gestational age of 8 weeks presents to the emergency department with persistent nausea and vomiting for the past week. She has lost 5 pounds in 7 days. She is lightheaded and has been unable to tolerate any liquid or food. She reports no diarrhea, dysuria, fever, or vaginal bleeding or discharge. Her T is 98.1°F, BP is 95/65 mm Hg, HR is 110 bpm, RR is 18/min, and oxygen saturation level is 98% on room air. She has decreased urine output with dark colored urine. Urinalysis is positive for ketones. Her serum blood urea nitrogen is 28 mg/dL, creatinine is 1.1 mg/dL, aspartate aminotransferase is 35 U/L, alanine transaminase is 39 U/L, serum bilirubin is 0.5 mg/dL, platelet count is 180,000 cells/microL, magnesium is 2.0 mg/dL, and potassium is 2.7 mmol/L. On physical examination, she has dry mucous membranes. Her abdomen is soft and nontender, and the psoas sign is negative. What is the most likely diagnosis?

Hyperemesis gravidarum develops at about 5 weeks, peaks at about 10 weeks, and usually resolves by 16-18 weeks of gestation. Patients may present with signs of dehydration with ketonuria, hypokalemia, and contraction alkalosis. Treatment typically consists of IV fluids, antiemetics, and vitamins.

A 29-year-old woman has been trying to get pregnant for 7 years. She has never been pregnant before. A previous physical exam and lab work were all within normal limits. An ovulation kit confirms that she is ovulating. Her fiancé has fathered two children and recently had a semen analysis that indicated normal functioning sperm. Which of the following is the most appropriate test of choice to evaluate this patient?

Hysterosalpingogram Assessment of fallopian tube patency or laparoscopy (if history strongly suggests prior tubal damage). Evaluation of the male partner is an integral part of the infertility workup and should coincide with the female partner's evaluation Infertility is defined as failure of conception after how long trying to conceive? Answer: One year.

A 30-year-old woman presents with fever and abdominal pain. She is three days postpartum after cesarean section. Physical examination reveals lower abdominal tenderness to palpation and foul smelling vaginal discharge. What management is indicated? A woman presents with fever and foul-smelling vaginal discharge three days after delivery of a full-term fetus. She is febrile, with uterine tenderness on pelvic exam. Which of the following is the strongest risk factor for postpartum endometritis? ACesarean section BInternal fetal monitoring CMultiple gestation DPrelabor rupture of membranes

IV Clindamycin and gentamicin Ampicillin-sulbactam What is an alternative regimen to treat endometritis? Answer: Intravenous ceftriaxone and intravenous metronidazole. dx:endometritis Patient will be postpartum, early-onset disease < 48 hours after delivery (C-section more common) Fever, abdominal pain, foul-smelling lochia PE will show uterine tenderness Labs will show leukocytosis Most common postpartum infection Cesarean section

A 17-year-old girl is seen in clinic due to complaints of excessive body hair. She denies taking any medication. She has irregular menses and denies sexual activity. On exam, her BMI is 31, with moderate hirsutism on upper lip and chest, moderate acne on her face, Tanner 5 breasts and pubic hair. The rest of her exam findings are normal. Which of the following is an expected laboratory finding?****

Increased luteinizing hormone dx: hyperandrogenism polycystic ovary syndrome (PCOS). PCOS is characterized by the triad of oligo-ovulation or anovulation Polycystic Ovary Syndrome (PCOS) Ovulatory dysfunction, hyperandrogenism, and polycystic ovaries Common PE findings: bilateral ovarian enlargement, acanthosis nigricans, high BMI Laboratory evaluation:Oligomenorrhea: hCG, FSH, TSH, prolactin Hyperandrogenism: total testosterone and sex hormone-binding globulin or bioavailable and free testosterone, morning 17-hydroxyprogesterone Metabolic disease screening: 2-hour oral glucose tolerance test, fasting lipid panel Commonly associated with insulin resistance, which may be the central etiology Treatment is combination low-dose oral contraceptive pills, lifestyle changes, metformin Most common cause of infertility Letrozole is first-line therapy for ovulation induction

Which of the following fetal heart tracings patterns is indicative of uteroplacental insufficiency?

Late decelerations Fetal Heart Rate: Mnemonic VEAL CHOP Variable: cord compression Early: head compression Accelerations: okay Late: placental insufficiency

A 23-year-old nulligravida comes to your office for contraception counseling. She has a seizure disorder that is well controlled on carbamazepine. She is a nonsmoker and has no other medical problems or complaints. She is currently in a relationship and does not want to get pregnant in the next several years. Which one of the following contraceptives is the most appropriate?

Levonorgestrel intrauterine device Certain antiepileptic drugs induce hepatic metabolism of estrogen (carbamazepine, oxcarbazepine, phenobarbital, phenytoin, and topiramate). This can potentially lead to failure of any contraceptive that contains estrogen.

Which of the following can help reduce the risk of developing lactational mastitis? A 32-year-old woman presents to the office with one day of fever, malaise, and left breast pain. She has a two-day old infant who she is trying to exclusively breastfeed. What breastfeeding practice in her daily routine is most likely to increase the risk of mastitis? A breastfeeding 28-year-old woman presents for evaluation of a painful right breast "lump" 3 weeks after delivery of a healthy newborn. Examination reveals localized erythematous edema of the right breast, a 7/10 painfully palpable discrete induration, and thick yellow nipple discharge. The nipple and areola are not excoriated. The left breast is unremarkable. You refer the patient for a drainage procedure. In the meantime, which of the following is the most appropriate initial plan?

Lubricating ointment for excoriations and cracking Long periods of time between feedings The main cause of mastitis is clogged milk ducts leading to inefficient drainage of milk during breastfeeding, Start antibiotics and continue breastfeeding with either breast First-line antibiotic choices include dicloxacillin, cephalexin, and clindamycin. Continue breastfeeding to decrease the chance of the mastitis progressing to a breast abscess

A 29-year-old woman who is one week postpartum following a pregnancy complicated by preeclampsia with delivery of a full-term infant is brought in by emergency medical services with an ongoing generalized tonic-clonic seizure. Which of the following medications should be administered first?****

Magnesium sulfate dx:eclampsia Lorazepam (B) and phenobarbital (D) are second- and third-line choices, What is the first clinical manifestation of magnesium toxicity? Answer: Loss of deep tendon reflexes.

A 32-year-old G1P0 woman presents to labor and delivery at 37 weeks gestation for elevated blood pressure and proteinuria noted in her obstetrician's office. The patient endorses a mild temporal headache and nausea. Blood pressure is found to be 168/102 mm Hg. On physical examination, there is edema of bilateral hands and hyperreflexia. On laboratory studies, the patient has 4 protein on urine dipstick. There is no evidence of hemolytic anemia or thrombocytopenia, but liver enzymes are elevated. In addition to urgently preparing for delivery, which of the following would be the most appropriate next step in the patient's management? A 21-year-old woman G1P0 at 35 weeks gestation presents with headache, blurry vision, and shortness of breath. Vital signs include a blood pressure of 195/110 mm Hg, heart rate of 90 beats per minute, respiratory rate of 21 breaths per minute, oral temperature of 37.1℃, and oxygen saturation of 90% on room air. Urinalysis reveals 3+ protein. A chest radiograph reveals pulmonary edema. In addition to magnesium, which of the following medications should be administered to reduce blood pressure? According to the United States Preventive Services Task Force (USPSTF), which of the following is considered to put a pregnant woman at high risk for developing preeclampsia? Which of the following is recommended to reduce the risk of preeclampsia in this pregnancy?

Magnesium sulfate for seizure prophylaxis dx: preeclampsia and hypertension should be treated with intravenous labetalol or hydralazine. Complications include eclampsia (development of seizures) and HELLP syndrome (hemolytic anemia, elevated liver enzymes, and low platelets). Clinical presentation may include headaches, visual disturbances, upper abdominal or substernal pain, dyspnea, or altered mental status. Hydralazine dx: severe preeclampsia. Autoimmune disease Aspirin

A 25-year-old woman presents to the emergency department with abdominal pain in the lower left quadrant. An ultrasound shows an ectopic pregnancy. Which of the following make her a candidate for methotrexate therapy?

Mass size less than 4 cm be hemodynamically stable, have a serum beta-human chorionic gonadotropin (hCG) concentration ≤ 5000 IU/L, and no fetal cardiac activity will be detected. dx: ectopic pregnancy

A 36-year-old G2P1002 woman who is 38 weeks pregnant presents in active labor. Her vital signs are a HR of 92 bpm, RR of 19/min, BP of 124/86 mm Hg, T of 97.8°F, SpO2 of 99% on room air, and BMI of 29.6 kg/m2. During delivery, the fetal head retracts into the perineum after expulsion, and the anterior shoulder does not deliver. Which of the following is the greatest risk factor for this presentation?

Maternal type 2 diabetes mellitus dx: shoulder dystocia, which is an obstetric emergency. . The initial approach in the management of shoulder dystocia is performing the McRoberts maneuver,which widens the pelvic outlet by hyperflexing the maternal hips.

During the menstruation section of a gynecologic history, you determine your 36-year-old patient's menstrual periods are increasing in length from 4 days to 8 days, even though she still cycles every 28 days. She also reports more blood flow than typical during the first 3 days of these new 8-day periods. Which of the following terms correctly defines this abnormal uterine bleeding?**************** A thin, 19-year old woman presents with abnormal vaginal bleeding stating that her menstrual cycle interval has shortened to 18 days. She reports normal flow and that her cycle lasts for 3-4 days. She has been under a lot of stress. What is this pattern of bleeding called?

Menorrhagia is defined as excessive bleeding (> 80 mL per cycle) or menstruation > 7 days, ************* Polymenorrhea occurs when cycles are occurring at intervals less than 21 days often caused by stress, excessive exercise, hormonal imbalance, endometriosis, perimenopause, fibroids, and sexually transmitted infections.

A 22-year-old woman at 36 weeks gestation presents with complaints of feeling a sudden gush of water coming from her vagina. She has had regular obstetrical follow-up and a normally progressing pregnancy. Which of the following is the most appropriate next step in management?

Microscopic evaluation of vaginal fluid dx:

A 36-year-old G1P0 woman at 9 weeks gestation presents to your office complaining of nausea and vomiting. She also reports mild vaginal bleeding. Pelvic exam shows a closed cervical os and dark blood in the posterior vault. The size of the uterus is felt to be larger than expected. Her beta-hCG level is 500,000 IU/L. A transvaginal ultrasound reveals a central heterogeneous mass with many discrete anechoic spaces. Which of the following is the most likely diagnosis? An 18-year-old G1P0 woman at 16 weeks gestation presents with vaginal bleeding. She has had no prenatal care. Vital signs are unremarkable and physical examination only reveals a small amount of blood in the vaginal vault. A transvaginal ultrasound is performed as seen above. What is the next best step in management of this patient?

Molar pregnancy Dietary deficiency of which vitamin is associated with an increased risk for complete mole? Answer: Vitamin A. PE will show uterine size that is larger than expected for dates Labs will show beta-hCG higher than expected for dates Diagnosis is made by ultrasound showing snowstorm or bag of grapes appearance Treatment is dilation and curettage New-onset hypertension < 20 weeks gestation: suspect molar pregnancy Obstetrics consultation dx:Molar pregnancy

A 22-year-old woman presents to your office with a complaint of cramping pelvic pain during her menses. She does not like to take medication. She has tried a heating pad, exercise, and multivitamins without relief of her symptoms. Which of the following is the most appropriate therapy? A 15-year-old girl presents for severe lower abdominal pain that occurs for the first two days of her menstrual cycle each month. She has associated nausea and diarrhea. She denies menorrhagia. She had a normal menarche at age 13 and is not sexually active. Abdominal and pelvic exams are normal. Which of the following is the initial best step in management?

Naproxen NSAIDor acetaminophen second line : estrogen-progestin contraceptives dx: Dysmenorrhea Primary Dysmenorrhea Pain starts 1 or 2 days before menses Pain is only related to menstrual cycle ↑ PGF2alpha → ↑ uterine contractions Pain management: NSAIDs (first line) or acetaminophen Hormonal therapy: estrogen-progestin contraceptives

A 67-year-old woman with no significant medical history presents to her gynecologist's office for her routine annual exam. The patient's last Pap smear and HPV cotesting were negative 2 years ago. She has not had any previous abnormal Pap smears. The patient smokes tobacco daily and uses alcohol socially. She continues to be sexually active with her partner of 33 years. Her vital signs are a HR of 74 bpm, RR of 17/min, SpO2 of 99% on room air, BP of 128/86 mm Hg, T of 98.1°F, and BMI of 33.6 kg/m2. On physical exam, her vaginal canal is slightly atrophic, the cervix visualized without scarring or erythema, and there is no tenderness on bimanual exam. What is the most appropriate recommendation for cervical cancer screening for this patient?

No further testing According to the United States Preventative Services Task Force (USPSTF), cervical cancer screening should be done between 21 and 65 years of age, regardless of the patient's sexual activity and HPV vaccine status.

A 55-year-old woman presents to the gynecology clinic with a 3-month history of gradual progressive dyspnea, rapid weight gain, and abdominal fullness. She also reports urinary frequency and urgency. Her last menstrual period was 5 years ago. Her T is 99.1°F, BP is 123/79 mm Hg, HR is 69 bpm, RR is 18/min, and oxygen saturation level is 98% on room air. Her urinary analysis is normal. Her physical examination reveals a distended abdomen with shifting dullness, and pelvic exam reveals a left adnexal mass. Which of the following would have been an appropriate recommended screening test for her condition before her symptoms started?

No screening recommended dx:Ovarian cancer Adnexal mass Most common histologic type is epithelial carcinomaTumor marker: CA 125 Rule out germ cell tumors in patients < 30 years old with tumor markers such as hCG and AFP The most common cause of gynecologic death Routine screening not recommended (lack of benefit) Where do most cases of ovarian cancer originate? Answer: Fallopian tubes. Which of the following is the most common cell type of ovarian cancer? AEpithelial

A 26-year-old woman presents to the emergency department with reports of sudden-onset severe right lower abdominal pain, beginning 2 hours ago after leaving the gym. She describes the pain as sharp and localized to her right lower abdomen. She states the pain is aggravated when sitting and reports no alleviating factors. She reports no significant medical history and takes no medications or supplements. Vital signs include a HR of 96 bpm, BP of 125/82 mm Hg, RR of 20/min, oxygen saturation of 98% on room air, and T of 98.6°F. On physical examination, she is tender to palpation in the right lower abdomen. Beta-human chorionic gonadotropin is negative, hemoglobin is 14.1 g/dL, WBC is 9,000/µL, and a transvaginal ultrasound reveals a right ovarian cyst with Doppler flow and a small amount of free fluid. What is the best next step for management of this condition? A previously healthy 25-year-old woman presents to the emergency department with complaints of right-sided lower abdominal pain and vaginal bleeding that began while she was running a 5K race. She has regular monthly cycles and expects to get her period in 3 days. Her blood pressure is 110/70 mm Hg, heart rate is 71 beats per minute, respiratory rate is 15 breaths per minute, and temperature is 98.9°F. Physical examination is normal except for mild tenderness to palpation of the right lower quadrant. An ultrasound shows no signs of inflammation and normal blood flow to both ovaries. A urine pregnancy test is negative. Which of the following is the most likely diagnosis?

Observation and reassurance dx: ruptured ovarian cyst sudden-onset unilateral lower abdominal pain due to blood stretching the ovarian cortex or irritating the abdomen. The pain is typically sharp and focal. Patients may also have increased pain while sitting due to irritation of the psoas muscle but if they are hemodynamically unstable, laparoscopy is the preferred surgical approach. In reproductive-aged women, most ovarian cysts spontaneously regress. Ovarian Cysts Follicular: most common ovarian mass, non-neoplastic, regress spontaneously Corpus luteum: most common ovarian mass in pregnancy, non-neoplastic, regress spontaneously Dermoid: teratoma Theca lutein: bilateral, ovarian enlargement Endometrioid: endometriosis within ovary, chocolate cyst Ultrasound Ruptured ovarian cyst sudden onset of one-sided pain in the lower abdomen. When pain occurs, it often begins during strenuous physical activity and patients may also experience light vaginal bleeding.

A 52-year-old woman with a history of unprovoked deep vein thrombosis at the age of 45 and treated with anticoagulation presents to her gynecologist with concerns for frequent hot flashes. She states her hot flashes mostly occur at night. The patient reports her last period was 2 years ago before having a hysterectomy for uterine fibroids. Her vital signs are a HR of 81 bpm, RR of 18/min, SpO2 of 99% on room air, BP of 126/82 mm Hg, T of 98.8°F, and BMI of 31.6 kg/m2. Her physical exam is normal. What is the best next step in management for this patient's presentation?*** You have been monitoring a 52-year-old perimenopausal woman's hot flashes. She has not had a hysterectomy. Her symptoms have been so mild that she does not require medication. However, for the past 2 months, her hot flashes have increased in frequency, duration, and intensity. She is now asking for a medication. Which of the following is the most appropriate for medical management of her moderate to severe symptoms? A 52-year-old woman presents to the clinic complaining of hot flashes and difficulty sleeping through the night. She states her last menstrual period was over 12 months ago. Which of the following laboratory results would be expected?

Oral paroxetine dx: menopause with hot flashes Estrogen plus progestin dx main symptom of a hot flash Risk factors include tobacco use and obesity. When symptoms become moderate to severe, estrogen therapy is recommended, either oral or transdermal. In women who have not had a hysterectomy, estrogen plus progestin is recommended to prevent endometrial hyperplasia. Elevated follicle-stimulating hormone dx: Menopause

A 19-year-old woman presents to the emergency department with pelvic pain that has been worsening over the last four days. She reports associated nausea and several episodes of emesis today. Vital signs include blood pressure of 113/68 mm Hg, pulse of 112 beats per minute, and oral temperature of 38.2°C. Physical exam reveals cervical motion tenderness and fullness in the left adnexa. Urine pregnancy test is negative. Which of the following would be the most appropriate next step in managing this patient? ABegin oral metronidazole and azithromycin BEmergent gynecology consult COrder a complete blood count and urinalysis DOrder a pelvic ultrasound A 32-year-old woman presents with fever and lower abdominal pain. She has a history of pelvic inflammatory disease. Her vitals are T 38.4°C, HR 133 bpm, and BP 101/60 mm Hg. On examination, the patient is toxic appearing and has marked lower abdominal tenderness to palpation with rebound and guarding. Pelvic examination reveals cervical motion tenderness, scant discharge, and left adnexal tenderness. The patient's urine beta-hCG is negative. A transvaginal ultrasound is performed and reveals a complex cystic, thick-walled, well-defined mass in the left adnexa. Which of the following is the most appropriate next step in management? A 21-year-old woman presents with pelvic pain that has been worsening over the last two days. She reports vaginal discharge, nausea, and chills for the last week. Physical exam reveals thin, copious vaginal discharge, cervical motion tenderness, and a mass in the left adnexa. Pregnancy test is negative. A pelvic ultrasound reveals a complex multiloculated left adnexal mass. Which of the following antibiotic regimens represents the most appropriate pharmacotherapy? AAmpicillin/sulbactam and clindamycin BCefoxitin and doxycycline CCeftriaxone and azithromycin DVancomycin

Order a pelvic ultrasound dx: tubo-ovarian abscess (TOA) and a pelvic ultrasound is the radiographic modality of choice to evaluate adnexal mass or fullness. TOA is often secondary to pelvic inflammatory disease (PID) Tubo-Ovarian Abscess History of pelvic inflammatory disease (PID) Lower abdominal pain, fever, vaginal discharge PE will show unilateral adnexal tenderness Diagnosis is made by ultrasound Most commonly caused by a complication of pelvic inflammatory disease Treatment is intravenous antibiotics, surgical drainage, or both Begin intravenous antibiotics and admit for possible drainage dx: tubo-ovarian abscess (TOA) Cefoxitin and doxycycline dx: tubo-ovarian abscess (TOA)

A 24-year-old woman comes to the clinic complaining of lower abdominal and pelvic pain. She says the pain has been ongoing for 3 to 4 weeks and is exacerbated when she has sexual intercourse with her boyfriend. She has had four sexual partners in the past year and only uses oral contraceptive pills for birth control. She also has dysuria and dysmenorrhea. Physical examination shows diffuse tenderness in the lower abdominal quadrants and cervical discharge. Which of the following is the most likely diagnosis?

Pelvic inflammatory disease dysuria, and dysmenorrhea are classical symptoms associated with pelvic inflammatory disease (PID). Pelvic Inflammatory Disease (PID) History of multiple sexual partners or unprotected intercourse Lower abdominal pain, cervical motion tenderness, painful sexual intercourse PE will show mucopurulent cervical discharge Most commonly caused by Chlamydia trachomatis Outpatient treatment is ceftriaxone + doxycycline + metronidazole Fitz-Hugh-Curtis syndrome: perihepatitis + PID

A 39-year-old woman in her third trimester presents with 2 days of bloody "spotting" on her underwear. This is her third pregnancy, which thus far has been uncomplicated. Her initial delivery was vaginal, and her second delivery was via cesarean section. She is currently sexually active and has a history of trichomoniasis. She denies pelvic pain. Laboratory examination reveals hematocrit of 32%, white blood cell count of 10,000/µL, platelet count of 260,000/µL, international normalized ratio of 1.1, and activated partial thromboplastin time of 32 seconds. Pelvic examination shows a nonerythematous cervix with clear mucus. Which of the following is the most likely diagnosis?

Placenta previa

A 35-year-old G2P1 woman at 28 weeks gestation presents to the Emergency Department with abdominal pain, continuous uterine contractions, and decreased fetal movement. She has a history of hypertension and endorses a 1.5 pack per day smoking history. She denies any vaginal bleeding or rush of water. Her vital signs upon arrival are T 37.2°C, HR 130, BP 80/50, RR 22. Fibrinogen is 200 mg/dL. A fetal heart rate by bedside Doppler is 100. Physical exam reveals a rigid, tender uterus. An ultrasound shows a normally implanted posterior placenta with diffuse thickening. Which of the following is the most likely diagnosis? Which of the following is an adverse pregnancy outcome due to maternal smoking? An 18-year-old woman in her third trimester presents with acute onset of significant pelvic pain and blood per vagina. Thus far, her pregnancy has been normal. Her past medical history is significant for hypertension, asthma, and recreational cocaine use. Examination reveals a tender, extremely tense uterus. Which of the following is the most likely diagnosis? ------------------------------------------------------- A 19-year-old G1P0 woman at 26 weeks presents with abdominal pain after being involved in a motor vehicle collision. External pelvic examination reveals vaginal bleeding. Which of the following is true regarding this presentation? Which of the following laboratory abnormalities is consistent with the most likely diagnosis? Hypofibrinogenemia

Placental abruption Placental Abruption Patient will be in third trimester History of hypertension, trauma, or cocaine use Painful vaginal bleeding Labs will show hypofibrinogenemia Tx: fetal monitoring, hemodynamic stabilization, delivery ================================= dx:Placental Abruption Emergent fetal monitoring and obstetric consultation are required Hypofibrinogenemia

A 15-year-old girl presents with generalized abdominopelvic pain that occurs every month after her regular, nonpainful menses. The pain, which is associated with headaches, bloating, and depressed mood, begins 18 days after the last day of menstruation. She also reports cyclic ankle swelling but does not report a history of increased salt intake. Which of the following is the most likely diagnosis?

Premenstrual syndrome PMDD represents a more severe disorder in which the symptoms result in significant distress or interference in usual activities.

A 14-year-old girl presents to clinic with some frustration over never having a menstrual period. She is short in stature and has Tanner stage 2 breast development. As you begin a gynecological exam, you realize you cannot pass a speculum into the vagina. Which of the following is the most likely diagnosis? A 14-year-old girl has been diagnosed with primary amenorrhea. Pelvic ultrasound reveals an absent uterus. Genetic testing confirms a 46,XX karyotype. Which of the following is the most likely diagnosis?

Primary amenorrhea # abnormal hymen in. this patient absence of menarche by age 15 in a woman with normal growth and secondary sexual development or age 13 in a woman without normal growth and secondary sexual development. most common cause of primary amenorrhea is caused by gonadal dysgenesis due to a chromosome abnormality, while other causes include hypothalamic disease, pituitary disease, abnormal hymen (as in the patient above) or vagina development, or uterine agenesis. Müllerian dysgenesis

A 34-year-old G5P4 woman at 24 weeks gestation presents to the emergency department with vaginal bleeding. A transabdominal ultrasound done in the emergency department shows the placenta overlying the cervical os. Which of the following is a risk factor for this condition?

Prior dilation and curettage dx Placenta previa Risk factors include previous placenta previa, multiple gestation, multiparity, prior cesarean section, and previous intrauterine surgery, including dilation and curettage. Any patient with active-phase labor arrest requires cesarean delivery.

In the clinic, you are evaluating a 15-year-old girl who is complaining of increased vaginal discharge. She had her menarche at 12 years of age and since then has had irregular periods. She uses tampons during her menses. She notes thick yellowish vaginal discharge in between her periods. She denies burning, pain, or pruritus. She started to be sexually active for the past month with one partner. She uses condoms for contraception. On examination, you note Tanner 5 pubic hair and white discharge. You examine the discharge and obtain pH of 4, with negative whiff test and absence of clue cells on microscopy. You perform a pregnancy test, which is negative. Which of the following is the next best step? 19-year-old woman presents to your clinic with concerns about increased, thicker vaginal discharge over the past 2 days. Her last menstrual period began 12 days ago. Her concern relates to her recent decision to begin having intercourse with her boyfriend. He is using condoms, but the condom broke 3 days ago. Vital signs are BP of 118/85 mm Hg, HR of 75 bpm, RR of 15/min, T of 98.7°F, and BMI of 22 kg/m2. No erythema is seen at the vulva, and the cervix appears pale pink with clear mucus at the cervical os. Upon bimanual exam, the uterus is normal in size, and the adnexa are somewhat tender. What is the best explanation for her discharge?

Provide her with reassurance dx: physiologic leukorrhea Ovulation

A 23-year-old G1P0 woman presents to the emergency department with vaginal bleeding. She is 8 weeks and 2 days pregnant according to an outpatient ultrasound she had last week showing an intrauterine pregnancy with a detectable fetal heart rate. She describes the bleeding as lighter than a normal period and reports she is not passing any clots or tissue. Vital signs include a HR of 80 bpm, BP of 120/80 mm Hg, RR of 20/min, oxygen saturation of 98% on room air, and T of 98.6°F. Physical examination includes a closed cervical os with a small amount of blood in the vaginal vault. Laboratory studies include a hemoglobin of 13 g/dL, beta-human chorionic gonadotropin of 100,000 mIU/mL, and a blood type of A positive. Transvaginal ultrasound in the emergency department shows an intrauterine pregnancy with a detectable fetal heart rate and appropriate interval growth since the last ultrasound. Which of the following is the most appropriate next step in management?

Provide reassurance and outpatient follow up with ultrasound dx: Threatened abortion

30-year-old woman presents for follow-up after dilation and curettage for a partial hydatidiform mole. Which of the following laboratory studies is the best method for identifying persistent or recurrent gestational trophoblastic disease?

Quantitative beta-human chorionic gonadotropin dxGestational trophoblastic disease (GTD) Patient presents with nausea, vomiting, abdominal pain, and vaginal bleeding PE will show uterine size that is larger than expected for dates Labs will show beta-hCG higher than expected for dates Diagnosis is made by ultrasound showing snowstorm or bag of grapes appearance Treatment is dilation and curettage New-onset hypertension < 20 weeks gestation: suspect molar pregnancy

At her annual checkup, a 33-year-old patient mentions difficulty conceiving after 7 months of unprotected and appropriately timed intercourse. She and her husband each have a child from their previous marriages. She experienced menarche at age 12 and reports a 30-day menstrual cycle with premenstrual symptoms of breast tenderness and bloating. Vital signs are BP of 110/70 mm Hg, HR of 70 bpm, RR of 12/min, T of 97.0°F, and BMI of 24 kg/m2. Findings on her Pap test 3 years ago were normal. Her facial skin is without blemishes or noticeable hirsutism. The pelvic exam reveals a cervical os with a transverse slit. The bimanual exam reveals a normal-sized uterus and no pain at the adnexa. What is the best next step for the couple's difficulty conceiving?

Reassurance and no further workup

A 65-year-old woman is evaluated for a 2-cm right breast mass discovered on routine screening mammography. There is no palpable breast mass or lymphadenopathy. Fine needle aspiration reveals invasive ductal carcinoma. Which of the following is the most appropriate next step in management? APalliative care BRight breast lumpectomy CRight breast lumpectomy, sentinel lymph node biopsy and radiation DRight breast mastectomy

Right breast lumpectomy, sentinel lymph node biopsy and radiation ----------------------------------------- Right breast lumpectomy and sentinel lymph node biopsy and radiation are the most appropriate next steps in management. Breast conserving therapy with lumpectomy followed by radiation therapy represents the standard of care for most women with small invasive breast cancers. Breast conserving surgery consists of surgical removal of the tumor followed by moderate-dose radiation therapy to eradicate any microscopic residual disease. Overall survival of patients who received lumpectomy and radiation is equivalent to modified radical mastectomy but is associated with improved cosmetic results and less morbidity. Lymph node evaluation is also important for prognostic and treatment purposes. Sentinel lymph node evaluation is now the standard of care instead of axillary node dissection and is often done along with breast conservation therapy. This provides information on the axillary lymph node status needed to make important decisions regarding adjuvant chemotherapy. Right breast mastectomy (D) is indicated for patients who are not eligible for breast conservation such as those in whom complete excision is not technically possible with lumpectomy alone or those in whom radiation is contraindicated. Right breast lumpectomy (B) alone does not provide information on the axillary lymph node status needed to make important decisions regarding adjuvant chemotherapy or eradicate microscopic residual disease with radiation and, therefore, is not the best next step in management. Palliative care (A) is not appropriate as this breast cancer is small in size and likely early-stage invasive ductal carcinoma with multiple treatment options.

pelvic inflammatory disease(PID)

Rx (outpatient) • Ceftriaxone 500 mg* IM PLUS • Doxycycline 100 mg PO BID x 14 days, WITH • Metronidazole 500 mg PO BID x 14 days Rx (inpatient) • Cefotetan 2 gm IV q 12 hours OR cefoxitin 2 gm IV q 6 hours PLUS • Doxycycline 100 mg PO/IV q 12 hours

Which of the following combination of laboratory findings is most suggestive of hemolysis, elevated liver enzymes, and low platelet count syndrome of pregnancy?

Schistocytes, thrombocytopenia, and elevated aspartate aminotransferase and alanine aminotransferase greater than twice normal dx HELLP syndrome

A 24-year-old woman presents for an initial evaluation of cyclical symptoms of irritability, painful bloating, and depression. These symptoms occur regularly 4-5 days prior to the beginning of menstruation. During these few days, the patient typically has to miss work due to the "awful pain and mental clouding." Which one of the following treatment options do you recommend as first-line therapy?

Selective serotonin reuptake inhibitors dx: mild premenstrual syndrome (PMS) Premenstrual Syndrome 1-2 weeks prior to menstrual cycle Sleep disturbances, decreased focus, emotional lability, breast tenderness, or HA that resolves after menses begins Treatment is decreased caffeine intake, exercise, stress reduction, NSAIDs, SSRIs, OCPs Symptoms do not hinder personal or professional life (unlike premenstrual dysphoric disorder)

A 24-year-old woman who first saw you 4 years ago for persistent acne, for which you prescribed a topical steroid, now presents in your office discouraged after 12 months of trying to conceive. Her vital signs are a BP of 120/80 mm Hg, HR of 87 bpm, RR of 14/min, T of 98.6°F, and a BMI of 22 kg/m2. Her last period was 1 week ago, but her periods are irregular. Her acne is still present, and she states she no longer takes medication for it. You verify her current medications and perform a vaginal exam, which is normal. All other signs are normal. Urine test is negative for pregnancy. What laboratory test would be the best next step for the suspected diagnosis? What sonographic findings would help to confirm your suspected diagnosis? Which of the following is this patient at greatest risk for developing malignancy of? A 26-year-old obese woman with a 2-year history of increased acne, abnormal hair growth, and menstrual abnormalities presents to her obstetrician for an infertility workup. A pelvic ultrasound reveals enlarged cystic ovaries. She desires to become pregnant. Which of the following is the next best step in treatment?

Serum total testosterone dx:polycystic ovary syndrome (PCOS). signs of hyperandrogenism such as hirsutism and acne Rotterdam criteria (2003) are considered the gold standard. In this scheme, two of the following three criteria must be met: 1) evidence of hyperandrogenism, such as high serum total testosterone or hirsutism; 2) evidence of ovarian dysfunction, such as oligomenorrhea or amenorrhea and infertility; and 3) polycystic ovaries (more than 12 antral follicles per ovary) on ultrasound. String-of-pearl appearance on the ovaries Endometrium Weight reduction is the first line treatment for polycystic ovarian syndrome for women who desire to conceive Clomiphene citrate (A) may be the next line of treatment if weight loss does not lead to return of ovulation. letrozole has emerged as the first line pharmacologic agent for ovulation induction in PCOS.

An 18-year-old woman presents to the clinic for her annual preventive health examination. She is asymptomatic. There is a new palpable left-sided adnexal mass on pelvic exam. Which of the following imaging modalities is best to initially characterize the mass?

Sonography dx:mature cystic teratoma, or dermoid cyst, is benign and the most common ovarian neoplasm in the second and third decades of life. ovarian neoplasm are asymptomatic, but symptomatic patients may present with abdominal pressure or pain, bloating or urinary symptoms, and ovarian torsion is not uncommon as a result of a dermoid cyst. Pelvic ultrasound is typically the first-line imaging study used to characterize an ovarian neoplasm. The primary concern is if the ovarian neoplasm is malignant, and sonographic findings can reliably determine whether a neoplasm is almost certainly benign or if it could reasonably be malignant

A 31-year-old woman with a history of hypertension and asthma presents to her OB/GYN's office. The patient is planning a pregnancy within the next few months. She takes lisinopril daily and albuterol as needed and is not on any other medications. Her vital signs are a HR of 65 bpm, RR of 17/min, SpO2 of 99% on room air, BP of 118/80 mm Hg, T of 97.3°F, and BMI of 28.6 kg/m2. Her most recent laboratory findings consist of the following values: A1C of 5.9%, potassium of 3.8 mmol/L, and creatinine at 0.8 mg/dL. Which of the following is the most appropriate management for this patient's hypertension?

Switch to nifedipine ER The following medications are acceptable for the treatment of hypertension in pregnancy: methyldopa, nifedipine, and labetalol.

Which of the following vaccinations is considered safe in pregnancy?

Tdap contraindicated following Mumps, Measles, Rubella (MMR) (Live, attenuated) • Varicella (Live, attenuated) • Oral polio vaccine (OPV) • BCG (Live, attenuated)

A 21-year-old woman presents with acute pain in the right pelvis. Which of the following makes the diagnosis of ovarian torsion more likely?

Teratoma of the right ovary or dermoid cysts, carry a particularly high risk of torsion.

A 22-year-old woman with regular menstrual cycles presents with symptoms concordant with premenstrual syndrome. In evaluating the large differential of these symptoms, which one of the following serum laboratory tests is recommended?

Thyroid-stimulating hormone

A 65-year-old patient is diagnosed with stage III endometrial adenocarcinoma. You decide to refer her to a gynecologic oncologist. She asks you to explain the "best treatment available." In your counseling, you discuss which one of the following treatment options?

Total abdominal hysterectomy

A 58-year old postmenopausal woman presents with painless vaginal bleeding. Her last menses occurred 5 years ago. She reports that her Papanicolaou smears have always been normal; the last one was obtained a year ago. Which of the following is the next step in management?

Transvaginal ultrasound

An otherwise healthy 60-year-old postmenopausal woman presents with 4 days of vaginal bleeding. Vital signs are BP 135/70, HR 85, RR 14, and T 98°F. You perform a pelvic exam and note blood at the cervical os. You are suspicious for a malignancy. In addition to performing a Pap smear, which of the following is the most appropriate next diagnostic test?

Transvaginal ultrasound dx:Endometrial Cancer

A 31-year-old woman at 35-weeks gestation presents with brief painless, bright red vaginal bleeding. In addition to fetal monitoring, which of the following is the most important initial management? A 23-year-old G1P0 woman at 30 weeks gestation presents to the emergency department for vaginal bleeding. She has had no prior prenatal care. She reports that she is soaking through one pad every 8 hours, denies abdominal cramping or a gush of water and endorses good fetal movement. Her vital signs are T 37.2°C, HR 100, BP 105/70, RR 18. Her abdomen is soft, non-tender, and consistent with a 30 week pregnancy. A transabdominal ultrasound shows a viable intrauterine pregnancy and a placenta that completely overlies the cervical os. Which of the following investigations is indicated in this patient?

Transvaginal ultrasound is safe and more accurate than transabdominal ultrasound for diagnosis of placenta previa​, Type and screen

A 22-year-old woman presents with pelvic pain that has been worsening over the last three days and vaginal discharge for the last week. She has associated nausea and chills. Physical exam reveals cervical motion tenderness and a mass in the right adnexa. Pregnancy test is negative. A pelvic ultrasound reveals a complex multiloculated right adnexal mass. Which of the following is the most likely diagnosis?***

Tubo-ovarian abscess Tubo-Ovarian Abscess History of pelvic inflammatory disease (PID) Lower abdominal pain, fever, vaginal discharge PE will show unilateral adnexal tenderness Diagnosis is made by ultrasound Most commonly caused by a complication of pelvic inflammatory disease Treatment is intravenous antibiotics, surgical drainage, or both

A 28-year-old woman presents with a concern about "lumpy, tender breasts." She states that the upper, outer quadrants of both breasts develop multiple small masses off and on throughout the month. The physical exam does not show a discrete lump, skin changes of the breasts, or nipple discharge. Which of the following is the initial best test to investigate her condition?

Ultrasonography Fibrocystic Breast Changes Risk factors: women 30−50 years old Sx: intermittent breast pain and tenderness that peak before each menstruation Ultrasound may show dense, prominent, fibroglandular tissue with cysts but no discernible mass Most commonly caused by fluctuating hormone levels during menstrual cycles Treatment is well-fitting supportive bras, applying heat to the breasts, or over-the-counter pain relievers Most common lesion of the breastFibrocystic changes are generally benign and do not increase risk for breast cancer

What are the black box warnings of tamoxifen?

Uterine cancer, stroke, pulmonary embolism. A 62-year-old woman presents for her annual well woman exam. She wants to discuss her risk of endometrial cancer, given a personal history of polycystic ovarian syndrome and prolonged tamoxifen use during her 50s. Given these risk factors she should be cautioned that early endometrial cancer normally presents in which of the following manners? Abnormal vaginal bleeding A 62-year-old woman is concerned with the recent appearance of blood spotting on her underwear, especially since her last menstrual period was 8 years ago. She has never had a sexually transmitted infection, although she does have a history of recurrent urinary tract infections. Pelvic examination reveals atrophic vaginal mucosa without evidence of bleeding. Adnexal masses are not palpable. Vulvar and urethral examination is unremarkable. Stool guaiac testing is negative. Which of the following is the most likely diagnosis? A 50-year-old woman presents to the office, reporting heavy menstrual periods for the past year that last for more than 10 days with about 21 days between each cycle. Her medical history includes age of menarche at 12 years, 9 years of oral contraceptive pill use, and one abnormal Papanicolaou smear at age 19 but with a negative HPV test at the time. Follow-up Papanicolaou smears have been negative. She has been pregnant four times and has had four live births. Her temperature is 97.2°F, blood pressure is 138/84 mm Hg, heart rate is 96 bpm, and respiratory rate is 22/minute. Her body mass index is 34 kg/m2. She is scheduled for an endometrial biopsy, and the patient is diagnosed with endometrial adenocarcinoma type 1. Which of the following is the most likely risk factor for this condition? Body mass index What is the most common type of endometrial cancer? Answer: Adenocarcinoma.

A 32-year-old African American woman presents to the outpatient office complaining of constant pelvic pressure with low back pain for several years. Her menstrual cycles are described as heavy and prolonged. Last menstrual period was ten days ago. Obstetrical history is significant for one full term pregnancy and two subsequent miscarriages. Gynecological history is significant for first menarche at the age of nine years. Family history is negative for gynecological malignancy. Physical exam reveals a firm, mobile, enlarged, nodular uterus. What is the most likely diagnosis? What is the diagnostic test of choice in a patient with a suspected leiomyoma?

Uterine leiomyoma Transabdominal and transvaginal ultrasound Uterine Fibroids (Leiomyoma) Common during reproductive-ages Menorrhagia and dysmenorrhea PE will show a enlarged, asymmetric, and nontender uterus Diagnosis is made by pelvic ultrasound Majority do not require surgical or medical treatment Severe cases: myomectomy (fertility can be preserved) or hysterectomy

A postmenopausal patient of yours is diagnosed with atrophic vaginitis and dyspareunia. She denies other menopausal complaints. Vaginal lubricants and moisturizers are not helping to decrease her symptoms. Which one of the following would be most the appropriate to prescribe next? A 65-year-old woman, whose last menstrual period was more than 10 years ago, complains of vaginal dryness and irritation. She denies any recent infections or sexual activity. She is afebrile with normal vital signs. Visual inspection of the vaginal canal reveals pale, dry, and shiny epithelium without frank discharge or superficial lesions. Which of the following conditions is the most likely diagnosis?

Vaginal estrogen Vaginal ring containing estradiol Genitourinary syndrome of menopause dx:atrophic vaginitis, can be quite bothersome and uncomfortable. They include irritation, burning, bleeding, dryness, and dyspareunia, Sx: dyspareunia, vulvar and vaginal dryness, bleeding, itching PE: pale, dry, shiny epithelium Caused by a decrease in estrogen Tx: lubricants, moisturizers, topical estrogen

A 22-year-old woman with no prior medical history presents with pain and swelling to the vulva. On examination, you notice an area of swelling with induration and central fluctuance at the 8 o'clock position. What is the best next step in management? A 45-year-old G0P0 woman presents to the office for a routine annual examination. Her medical history is significant for diabetes mellitus, controlled well on metformin, and hypertension, controlled well on losartan. She is not sexually active and reports no previous sexually transmitted infections. Vital signs include HR of 69 bpm, BP of 130/85 mm Hg, RR of 15/min, oxygen saturation of 99% on room air, and T of 98.7°F. Physical examination reveals a painless 2 cm round nodule on the right posterior vaginal orifice. What is the most likely diagnosis?

Word catheter placement for 4-6 weeks xd: Bartholin abscess ↑ pain with sitting or walking a pea-sized mucous secreting gland located on each side of the labia minora in the 4 and 8 o'clock positions. E. coli is most prevalent in cultures. Rx: I&D on mucosal surface, Word catheter Treatment of the abscess is with incision and drainage. After the loculations are broken up, and all contents of the abscess have been expressed, either gauze packing or a Word catheter is placed. Gauze packing is removed after 24 to 48 hours, and patients may require marsupialization at a later date. The Word catheter is a plastic catheter with a balloon on the end that is filled with 2-4 mL of water and expanded within the abscess cavity. It is important that the incision is made only slightly larger than the opening required for catheter placement. The catheter remains in place for 4-6 weeks, and patients are instructed to abstain from vaginal intercourse. This duration is required for epithelialization of the tract and the formation of a small fistula tract that will allow appropriate drainage without obstruction.


Ensembles d'études connexes

A.2.1 Linux Pro Domain 1: System Administration and Configuration

View Set

U.S. History 3-6-14 Study this for test next week

View Set

Microbiology Chapter 20:Antimicrobial Drugs

View Set

Ch.7 Political Participation: Activating the Popular Will

View Set

psych 220 wakefulness and sleep & internal regulation

View Set